esercizi limiti di due variabili reali

In questa sezione, approfondiremo tre esercizi svolti sui limiti in due variabili. Comprendere i limiti è fondamentale in matematica, in quanto rappresentano il concetto di avvicinamento infinitesimo di una funzione a un determinato punto. Questi esercizi ci aiuteranno a consolidare le nostre conoscenze su questo argomento cruciale.

Calcolare i seguenti limiti di due variabili reali con sostituzione y=mx

Vogliamo calcolare i seguenti limiti

1) $\underset{\left( x,y \right)\to \left( 0,0 \right)}{\mathop{\lim }}\,\frac{xy}{x+y}$ ,

2) $\underset{\left( x,y \right)\to \left( 0,0 \right)}{\mathop{\lim }}\,\frac{x{{y}^{2}}}{x+y}$

3) $\underset{\left( x,y \right)\to \left( 0,0 \right)}{\mathop{\lim }}\,\frac{x³ \cdot y²}{{{x}^{4}}+{{y}^{4}}}$

Proviamo a risolvere l’esercizio sui limiti in due variabili con sostituzione y=mx.

fascio di rette passanti per l'origine usato per gli Esercizi svolti sui limiti in due variabili
Sostituzione y=mx nei limiti di due variabili reali (fascio di rette passanti per l’origine degli assi cartesiani)

1. Soluzione del primo limite di funzione due variabili reali

$\underset{\left( x,y \right)\to \left( 0,0 \right)}{\mathop{\lim }}\,\frac{xy}{x+y}$

Per questo limite proviamo ad utilizzare la sostituzione y=mx, e vediamo come arrivare alla soluzione ed in particolare, dimostrare se esiste o non esiste il limite.

Osserviamo innanzi tutto che $y=mx$ è una retta passante per l’origine e al variare di m cambia la pendenza e quindi la direzione con cui essa tende verso l’origine degli assi, quando $x \to 0$ e di conseguenza anche $y \to 0$

Ogni valore di m rappresenta una direzioneIl limite esiste, se il risultato non dipende da m, tenendo presente che m può tendere a $\pm \infty $ nel caso in cui la retta tende a coincidere con l’asse y.

Facciamo la sostituzione nel limite e si ha:

$\underset{x\to 0}{\mathop{\lim }}\,\frac{m{{x}^{2}}}{x+mx}$

$=\underset{x\to 0}{\mathop{\lim }}\,{x} \cdot \frac{m}{1+m}$

Dove $m$ è una variabile che può assumere qualsiasi valore reale.

In questo caso otteniamo il prodotto di due fattori:

  • Il primo fattore è $x$ e tende a zero quando $x\to 0$
  • Il secondo fattore è $\frac{m}{1+m}$ è una quantità che dipende dal parametro m, e non risulta limitata notando infatti che:   

$\underset{m\to -1}{\mathop{\lim }}\,\frac{m}{1+m} = \infty$

 Pertanto, questa sostituzione non ha portato a niente, in quanto ci troviamo di fronte ad una forma indeterminata del tipo $0 \cdot \infty$.

Possiamo notare che la forma a cui ci siamo portati è una forma indeterminata, essendo composto da una quantità che tende a zero per una quantità finita, quando si tende all’origine degli assi percorrendo una retta con coefficiente angolare $m=-1$. Ma forma indeterminata non vuol dire che il limite non esiste, vuol dire semplicemente che con questo metodo non ci ha portato a nessuna conclusione! Dobbiamo cambiare metodo:

Possiamo considerare altre curve passanti per l’origine degli assi, come $ y = x^2 $ o $ y = -\sin (x) $ e valutare il limite lungo di esse. Se otteniamo valori diversi del limite lungo curve diverse, allora il limite generale non esiste.

Proviamo con $ y = x^2 $, il limite diventa:

$\underset{x\to 0}{\mathop{\lim }}\,\frac{x³}{x+x²} = 0$

Proviamo con $ y = -\sin (x) $, il limite diventa:

$ \lim_{x \to 0} \frac{x(-\sin(x))}{x + (-\sin(x))} $

Approssimo con Taylor numeratore e denominatore:

$ = \lim_{x \to 0} \frac{-x^2 +o(x²)}{x – x + \frac{x^3}{6} + o(x³)} $

A questo punto possiamo trascurare gli o-piccoli.

$ = \lim_{x \to 0} \frac{x^2}{\frac{x^3}{6}} $

$ = \lim_{x \to 0} \frac{6}{x} = \infty $

A questo punto possiamo finalmente concludere che il limite non esiste perché dipende da come ci si tende all’origine, e si andrebbe in contraddizione con il teorema di unicità del limite.

Ma come mai ho scelto proprio la curva $ y = -\sin (x) $? Semplice!! Perché siccome dalla sostituzione $y=m \cdot x$ ho notato che qualche problema si verificava quando $m=-1$ allora ho cercato una curva che si avvicinasse a zero in maniera molto simile a $y=-x$

2. Soluzione del secondo degli Esercizi svolti sui limiti in due variabili

Vogliamo calcolare il limite:

 $\underset{\left( x,y \right)\to \left( 0,0 \right)}{\mathop{\lim }}\,\frac{x{{y}^{2}}}{x+y}$

Similmente al caso precedente se facciamo la sostituzione con $y=m \cdot x$ ci ritroviamo di fronte ad una forma indeterminata $0 \cdot \infty$. quando $ m \to -1 $. Ma usando le stesse curve precedenti otteniamo una situazione simile:

Possiamo considerare altre curve passanti per l’origine degli assi, come $ y = x^2 $ o $ y = -\sin (x) $ e valutare il limite lungo di esse. Se otteniamo valori diversi del limite lungo curve diverse, allora il limite generale non esiste.

Proviamo con $ y = x^2 $, il limite diventa:

$\underset{x\to 0}{\mathop{\lim }}\,\frac{x⁴}{x+x²} = 0$

Proviamo con $ y = -\sin (x) $, il limite diventa:

$ \lim_{x \to 0} \frac{x(-\sin(x))}{x + (-\sin(x))} $

Approssimo con Taylor numeratore e denominatore:

$ = \lim_{x \to 0} \frac{-x^3}{x – x + \frac{x^3}{6} + o(x³)} $

A questo punto trascuriamo gli o-piccoli:

$ = \lim_{x \to 0} \frac{x^3}{\frac{x^3}{6}} $

$ = \lim_{x \to 0} \frac{6}{1} = 6 $

E anche in questo caso si va in contraddizione con il teorema di unicità del limite, e si può concludere che anche questo limite non esiste!

Traggo l’occasione per far notare che qualsiasi limite della forma $\underset{\left( x,y \right)\to \left( 0,0 \right)}{\mathop{\lim }}\,\frac{{x}^{n}{{y}^{m}}}{x+y}$ non esiste. Basta sperimentare con delle parametrizzazioni del tipo $y=-x+\alpha \cdot {x}^{m+n}$: provare per credere! E questo per sottolineare che a differenza di come l’intuito potrebbe suggerire, non conta che il numeratore è apparentemente di ordine anche molto più grande del denominatore, il limite può comunque non esistere.

3. Soluzione del terzo limite di funzione di due variabili

Vogliamo calcolare il limite:

 $\underset{\left( x,y \right)\to \left( 0,0 \right)}{\mathop{\lim }}\,\frac{x³ \cdot y²}{{{x}^{4}}+{{y}^{4}}}$

Risolviamo anche questo esercizio sui limiti in due variabili con sostituzione $y=mx$. Facciamo la sostituzione $y=mx$ e vediamo cosa succede:

 $\underset{\left( x,y \right)\to \left( 0,0 \right)}{\mathop{\lim }}\,\frac{x³ \cdot y²}{{{x}^{4}}+{{y}^{4}}}$

$=\underset{ x \to 0}{\mathop{\lim }}\,\frac{{{x}^{5}}{{m}^{4}}}{{{x}^{4}}+{{m}^{4}}{{x}^{4}}}$

$=\underset{\left( x,y \right)\to \left( 0,0 \right)}{\mathop{\lim }}\, x \cdot \frac{{{m}^{4}}}{1+{{m}^{4}}}$

dove $m$ è una variabile reale che può assumere qualsiasi valore

In questo caso otteniamo il prodotto di due fattori:

  • Il primo fattore è $x$ e tende a zero quando $x\to 0$
  • Il secondo fattore è $\frac{m⁴}{1+m⁴}$ è una quantità che dipende dal parametro m, ma in questo caso risulta limitata notando infatti che:   

$0 \leq \frac{m⁴}{1+m⁴}<1$

Essendo il denominatore sempre maggiore del numeratore. Questa volta non siamo più di fronte ad una forma indeterminata, ma di fronte ad una quantità limitata che moltiplica un fattore che tende a zero. Si può quindi concludere che il limite esiste e fa zero.

E con questo concludiamo la collana dei 3 Esercizi svolti sui limiti in due variabili. Ma non finisce qua, infatti se continui a navigare nel sito tramite i link in basso, troverai tanti altri esercizi svolti sui limiti in due variabili reali, quindi continua liberamente la tua navigazione per leggere altri esempi svolti e trovare nuovi spunti di riflessione e ragionamento, che meglio si adattano alle tue esigenze verso la preparazione dei tuoi esami universitari.

Ah dimenticavo hai un limite, ma ti manca il risultato? Perché non provi questo strumento fornito da WolframAlpha, non ti mostra i passaggi se non hai un abbonamento pro, ma comunque è molto utile per ottenere il risultato atteso: